2
$\begingroup$

Define $Z$ as a sort of zeta function defined over the powers of the non-trivial zeros $\rho$ of the Riemann zeta function (a meta-zeta function). So we have, $$Z(s) = \sum_{\rho} \frac{1}{\rho^s}$$ How can I prove that this sum converges absolutely for $\Re(s) >1$?


I am sure this function has been studied before, but I can't find any references on its convergence. I know that $Z(s)$ converges if the $S$ (below) converges. I was able to show that $S$ converges, but is this a valid proof, or the conventional proof? Thanks \begin{align} S &= \sum_{\gamma_n > 0} \frac{1}{\gamma_n^{\sigma}} = \int_{\gamma_1}^{\infty} \frac{1}{T^\sigma} \, dN(T) \\ &= \left[\frac{N(T)}{T^{\sigma}} \right]_{\gamma_1}^{\infty} - \int_{\gamma_1}^{\infty} N(T) \left( \frac{d}{dT} T^{- \sigma} \right) \, dT \\ &= \eta + \sigma \int_{\gamma_1}^{\infty} \frac{N(T)}{T^{\sigma+1}} \, dT \end{align}

where $\eta$ is some constant. Thus, the sum converges if the integral converges. Indeed, for the integral we may use the Riemann–von Mangoldt formula

$$\int_{\gamma_1}^{\infty} \frac{N(T)}{T^{\sigma + 1}} \, dT \sim \int_{\Omega}^{\infty} \frac{\frac{T}{2\pi} \log T}{T^{\sigma+1}} \, dT = \xi \int_{\Omega}^{\infty} \frac{\log T}{T^{\sigma}} \, dT$$

which converges if $\sigma > 1$ and diverges if $\sigma \le 1$.

$\endgroup$
3
  • 1
    $\begingroup$ When you know the zero-count is ${1\over 2\pi}T\log T$, you already know the convergence of your infinite sum in $\Re(s)>1$. $\endgroup$ Commented Oct 26 at 18:52
  • $\begingroup$ a related way is to show that the series converges iff $\sum 1/\gamma_n^s$ does (where $\rho_n=\beta_n+i\gamma_n, \gamma_n>0$) and note that the latter is a generalized Dirichlet series and then the formula for the abscissa of convergence (absolute as coefficients are nonnegative anyway) is $\limsup \frac{\log n}{\log \gamma_n}$ and that is easily seen to be $1$ $\endgroup$ Commented Oct 28 at 16:30
  • $\begingroup$ @Conrad, if you have time, could you post your comment as an answer. Thanks $\endgroup$ Commented 2 days ago

1 Answer 1

3
$\begingroup$

Let me expand my comment into a short answer; first since $|\rho_n|=|\sigma_n+i\gamma_n|\to \infty$ where we index the nontrivial roots by increasing $|\gamma_n|$, it is clear that we need $\Re s >0$ to have any hope of convergence.

Second since $0<\sigma_n<1$ then for any $a>0$ we have $|(\sigma_n+i\gamma_n)^{a}-(i\gamma_n)^a|=|\int_0^{\sigma_n} (x+i\gamma_n)^{a-1}dx/a| \le C_a|\gamma_n|^{a-1}$ so using that (for say $n \ge 2$ to avoid trivialities) $An/ \log n \le |\gamma_n| \le Bn/\log n$ (since up to level $|\gamma| \le m$ we have roughly $Cm \log m$ roots with negligible relative error) it follows that $$\sum \frac{1}{\rho^s}-\sum\frac{1}{(i\gamma_n)^s}$$ converges absolutely for $\Re s =a >0$ and hence the convergence of the original series is equivalent to the convergence of $\sum |\gamma_n|^{-s}=\sum e^{-s\log |\gamma_n}|$.

But this is a generalized Dirichlet series since $|\gamma_n| \to \infty$ and its abscissa of convergence and absolute convergence since the coefficients are $1$ hence positive, is given by the standard formula $$\alpha=\limsup_N\frac{\log A_N}{\log |\gamma_N|}$$ where $A_N$ is the summatory function of the coefficients of the series up to index $N$ so $A_N=N$ as all the coefficients are $1$ after all.

But $A N/\log N \le |\gamma_N| \le BN/\log N$ implies immediately that $\alpha=1$ so we are done!

$\endgroup$

You must log in to answer this question.

Start asking to get answers

Find the answer to your question by asking.

Ask question

Explore related questions

See similar questions with these tags.